Help Proving a Complex Laplace Transform

Click For Summary
The discussion centers on a homework problem involving the proof of a complex Laplace transform expressed in terms of variables x and t. The original poster is uncertain about how to handle the exponent and denominator in the function while attempting to use the definition of the Laplace transform. They express frustration with breaking the problem down and finding relevant information from existing tables. A response clarifies that the relevant equation is the definition of the Laplace transform and emphasizes that definitions do not require proof. The conversation highlights the need for understanding the application of the Laplace transform in this context.
EaglesFan7
Messages
2
Reaction score
0
Homework Statement
Hi, for a homework problem I have to prove this LaPlace transform, but I've never seen anything like this because it's in terms of x and t. I'm assuming x can just be left as constant but I'm not sure how to account for the exponent and the denominator. This definitely isn't something you can find on a table but I tried to break it down.
Relevant Equations
L(f(t)) = limit T ---> infinity of integral from 0 to T of e^(-st) * f(t)dt
So I could just try using the definition by taking the limit as T goes to infinity of ∫ from 0 to T of that entire function but that would be a mess. I tried breaking it down into separate pieces and seeing if I could use anything from the table but I honestly have no clue I'm really stuck. I'd appreciate any help, thank you guys.
 

Attachments

  • OPM1.PNG
    OPM1.PNG
    2.3 KB · Views: 291
Physics news on Phys.org
EaglesFan7 said:
Problem Statement: Hi, for a homework problem I have to prove this LaPlace transform, but I've never seen anything like this because it's in terms of x and t. I'm assuming x can just be left as constant but I'm not sure how to account for the exponent and the denominator. This definitely isn't something you can find on a table but I tried to break it down.
Relevant Equations: L(f(t)) = limit T ---> infinity of integral from 0 to T of e^(-st) * f(t)dt

So I could just try using the definition by taking the limit as T goes to infinity of ∫ from 0 to T of that entire function but that would be a mess. I tried breaking it down into separate pieces and seeing if I could use anything from the table but I honestly have no clue I'm really stuck. I'd appreciate any help, thank you guys.
What is there to prove? Your relevant equation is basically the definition of the Laplace transform of a function f, and you use definitions -- you don't prove them.

Here's your equation in a nicer format:
$$\mathcal L(f(t)) = \lim_{T \to \infty}\int_0^T e^{-st}f(t)dt$$
Click on the equation I wrote to see the LaTeX script I wrote.
Also, there's a link to our tutorial on LaTeX at the bottom left of the pane.
 
  • Like
Likes berkeman
Yes, here x is just an arbitrary constant.
 
Question: A clock's minute hand has length 4 and its hour hand has length 3. What is the distance between the tips at the moment when it is increasing most rapidly?(Putnam Exam Question) Answer: Making assumption that both the hands moves at constant angular velocities, the answer is ## \sqrt{7} .## But don't you think this assumption is somewhat doubtful and wrong?

Similar threads

  • · Replies 10 ·
Replies
10
Views
2K
  • · Replies 4 ·
Replies
4
Views
3K
  • · Replies 8 ·
Replies
8
Views
2K
  • · Replies 4 ·
Replies
4
Views
3K
Replies
9
Views
2K
  • · Replies 1 ·
Replies
1
Views
1K
Replies
5
Views
2K
Replies
3
Views
2K
  • · Replies 1 ·
Replies
1
Views
2K
Replies
5
Views
3K